LSAT and Law School Admissions Forum

Get expert LSAT preparation and law school admissions advice from PowerScore Test Preparation.

 Administrator
PowerScore Staff
  • PowerScore Staff
  • Posts: 8916
  • Joined: Feb 02, 2011
|
#88558
Complete Question Explanation
(The complete setup for this game can be found here: lsat/viewtopic.php?f=182&p=88548#p88548)

The correct answer choice is (C).

As with all List questions, simply apply the rules to the answer choices. In this game, the easiest approach is to apply the rules in this order: 2, 1, 4, 3.

Answer choice (A): This answer choice violates the third rule.

Answer choice (B): This answer choice violates the fourth rule.

Answer choice (C): This is the correct answer choice.

Answer choice (D): This answer violates the first rule.

Answer choice (E): This answer violates the second rule.
User avatar
 anonymous123
  • Posts: 1
  • Joined: Jul 19, 2023
|
#106041
How should I have approached this question?

I recall being told to find the first answer choice that COULD work and move on and in my initial scenario, I see F first in a FHGKLM setup...how is this wrong?

Why is C right and A wrong?
 Luke Haqq
PowerScore Staff
  • PowerScore Staff
  • Posts: 747
  • Joined: Apr 26, 2012
|
#106048
Hi anonymous123!

(C) is correct because that arrangement doesn't violate any of the rules, whereas the four other answer choices all involve arrangements that in some way violate the rules.

To speak to answer choice (A) specifically, that gives us the arrangement H,F,L,G,K,M. This arrangement violates the third rule: "If Figueroa's delivery is earlier than Malpighi's, then Leacock's delivery is earlier than Harris's." In the arrangement in (A), F is earlier than M, which should mean that L must be earlier than H, which isn't the case, thus this arrangement wouldn't work.

Get the most out of your LSAT Prep Plus subscription.

Analyze and track your performance with our Testing and Analytics Package.